What is the solution to X+9 = 24?
A. x = 33
B. x= 15
C. x= 18
D. x= 9​

Answers

Answer 1

Answer:

X+9=24

Or,x=24-9

:.x=15

Step-by-step explanation:

Answer 2

Answer:

B. x=15

Step-by-step explanation:

To find the solution to the equation, we must get x by itself on one side of the equation.

[tex]x+9=24[/tex]

9 is being added to x. The inverse of addition is subtraction. Subtract 9 from both sides of the equation.

[tex]x+9-9=24-9[/tex]

[tex]x=24-9[/tex]

[tex]x=15[/tex]

Let's check our solution. Plug 15 in for x.

[tex]x+9=24 (x=15)[/tex]

[tex]15+9=24[/tex]

[tex]24=24[/tex]

This checks out, so we know our solution is correct. The answer is B. x=15


Related Questions

Evaluate the expression for y=-1? 14+5y=

Answers

Answer:

The answer is 9

Step-by-step explanation:

14 + 5y

To solve the expression substitute the value of y that's - 1 into the expression

That's

14 + 5(-1)

= 14 - 5

= 9

Hope this helps you

Solve for x (x+4)/3 = 2.

a. x = -2

b. x=2

c. x = 2/3

d. x= -10/3​

Answers

Answer:

The answer is option B

Step-by-step explanation:

[tex] \frac{x + 4}{3} = 2[/tex]

To solve it first of all cross multiply

That's

x + 4 = 6

Move 4 to the right side of the equation

The sign changes to negative

That's

x = 6 - 4

We have the final answer as

x = 2

Hope this helps you

Solve the equation.

Answers

Multiple all by 7
2x +6=-2
2x = -2-6
2x = -8 /2
x = -4

Find the product: 12 x 3/5 =

Answers

Answer:

12 x 3/5 = 7 1/5

Step-by-step explanation:

12 x 3/5

Add 1 below 12 as a denominator to make it an improper fraction

= 12/1 x 3/5

Multiply numerators from both fractions as long as the denominators:

12 x 3 = 36

1 x 5 = 5

12/1 x 3/5 = 36/5

36/5 SIMPLIFIED IS 7 1/5

Hope this helps!

Answer:

36/5 = 7.2 = 7 1/5

Step-by-step explanation:

12 x 3/5

Change 12 into fraction form to make it easier.

12/1 x 3/5

Now multiply the numerators and the denominators.

12 x 3 = 36

1 x 5 = 5

12/1 x 3/5 = 36/5

If you don't want the answer as an improper fraction, 36/5 = 7.2 which is also equal to 7 1/5

a=5,and 5+z=14,so a+z=14

Answers

Answer:

Z=9

Step-by-step explanation:

Insert A into A+Z=14

5+z=14

Subtract 5 on both sides, to find Z.

-5     -5

z=9

Explain why this quadrilateral is not a parallelogram.

Answers

Answer:

A parallelogram has two sets of parallel sides. This quadrilateral only has on set of parallel sides, so therefore it cannot be a parallelogram.

does x represents Mount Everest height ?​

Answers

Answer:

Bro what do you mean to represent Mount Everest height by x but if you want to sove a maths sum than you can take any alphabet

Step-by-step explanation:

If you like my answer than please mark me brainliest thanks

find the common ratio of the geometric sequence 4,3,9/4

Answers

Answer:

3/4

Step-by-step explanation:

To find the common ratio, take the second term and divide by the first term

3/4

Check with the third and second terms

9/4 ÷3

9/4 *1/3= 3/4

The common ratio is 3/4

Find the degree, leading coefficient, and the constant term of the polynomial.

Answers

[tex] \LARGE{ \boxed{ \purple{ \rm{Answers;)}}}}[/tex]

☃️ Degree of the polynomial- The highest degree of any term in a polynomial. Here the highest degree is 5.

⇛ 4x⁴ + 5 + 6x⁵ - 2x(° of polynomial = 5)

☃️ Leading coefficient- The coefficient of the term having the highest degree of the polynomial. Here, the highest degree is 5 and the term is 6x

⇛ 4x⁴ + 5 + 6x⁵ - 2x (Leading coeff. = 6)

☃️ Constant term- It is the term having no coefficients, only a fixed real number. This remains constant in any value of polynomial.

⇛ 4x⁴ + 5 + 6x⁵ - 2x (Constant term = 5)

━━━━━━━━━━━━━━━━━━━━

What is the error in this problem

Answers

Answer:

10). m∠x = 47°

11). x = 30.96

Step-by-step explanation:

10). By applying Sine rule in the given triangle DEF,

   [tex]\frac{\text{SinF}}{\text{DE}}=\frac{\text{SinD}}{\text{EF}}[/tex]

   [tex]\frac{\text{Sinx}}{7}=\frac{\text{Sin110}}{9}[/tex]

   Sin(x) = [tex]\frac{7\times (\text{Sin110})}{9}[/tex]

   Sin(x) = 0.7309

   m∠x = [tex]\text{Sin}^{-1}(0.7309)[/tex]

   m∠x = 46.96°

   m∠x ≈ 47°

11). By applying Sine rule in ΔRST,

   [tex]\frac{\text{SinR}}{\text{ST}}=\frac{\text{SinT}}{\text{RS}}[/tex]

   [tex]\frac{\text{Sin120}}{35}=\frac{\text{Sin50}}{x}[/tex]

   x = [tex]\frac{35\times (\text{Sin50})}{\text{Sin120}}[/tex]

   x = 30.96   

What is the measure of B, in degrees?

Answers

Answer:

B = 32

Step-by-step explanation:

Since this is an isosceles triangle, C is also equal to 74 degrees

 the angles of a triangle add to 180

A + B+ C = 180

74+ B + 74 = 180

148 + B = 180

B = 180-148

B =32

what is the prime factorization of 7?

Answers

the prime factorization of 7 is 7

As Prime factorization is a process of writing all numbers as a product of primes then The prime factorization of number 7 is 7.

What is Number system?

A number system is defined as a system of writing to express numbers.

Prime factorization is a process of writing all numbers as a product of primes

The number 7 is a prime number, which means it is only divisible by 1 and itself.

Therefore, the prime factorization of 7 is simply 7 itself.

Hence, the prime factorization of number 7 is 7.

To learn more on Number system click:

https://brainly.com/question/22046046

#SPJ2

I will rate you brainliest Select the best description of what the LCM of a set of polynomials is. a.It is the quotient of all the factors of the polynomials. b.It is the common numerator of a rational expression. c. It is the product of the prime factors that are either unique to or shared by the polynomials. d. It is all the polynomials in the set.

Answers

Answer:

C. It is the product of the prime factors that are either unique to or shared by the polynomials.

Step-by-step explanation:

LCM of polynomials is:

=> Finding the factors of all the numbers and variable in the expression

=> Next, we multiply the unique numbers and the variable of the expression to find the LCM.

So, C is the correct answer.

The LCM of a set of polynomials  is the product of the prime factors that are either unique to or shared by the polynomials.  

What is LCM of polynomial?

To find the lowest common multiple (L.C.M.) of polynomials, we first find the factors of polynomials by the method of factorization and then adopt the same process of finding L.C.M.

Example : The L.C.M. of 4a2 - 25b2 and 6a2 + 15ab.

Factorizing 4a2 - 25b2 we get,

(2a)2 - (5b)2, by using the identity a2 - b2.

= (2a + 5b) (2a - 5b)

Also, factorizing 6a2 + 15ab by taking the common factor '3a', we get

= 3a(2a + 5b)

L.C.M.  is 3a(2a + 5b) (2a - 5b)

According to the question

The LCM of a set of polynomials is

 is the product of the prime factors that are either unique to or shared by the polynomials.  

(from above example we can see that )

Hence,  It is the product of the prime factors that are either unique to or shared by the polynomials.  

To know more about LCM of polynomial here :

https://brainly.com/question/26227783

# SPJ2

please help, will give brainliest for correct answer

Answers

ain't it just 3 for each one unless i'm missing something

What is the answer and how is this solved?

Answers

Answer:

Sum : 65

Step-by-step explanation:

In this notation, n is our starting value, and hence we start at 3 and go to 7. Given the set of values : { 3, 4, 5, 6, 7 }, we can substitute in our expression " 4n - 7 " for n and solve. The sum of these values is our solution.

4( 3 ) - 7 = 12 - 7 = 5,

4( 4 ) - 7 = 16 - 7 = 9,

4( 5 ) - 7 = 20 - 7 = 13,

Our remaining values for n = 6 and n = 7 must then be 17 and 21. This is predictable as we have an arithmetic series here, the common difference being 4. As you can see 9 - 5 = 4, 13 - 9 = 4, 17 - 13 = 4, 21 - 17 = 4.

Therefore we have the series { 5, 9, 13, 17, 21 }. This adds to an answer of 65.

The U.S. National Whitewater Center in Charlotte uses a pump station to provide the flow of water necessary to operate the rapids. The pump station contains 7 pumps, each with a capacity to deliver 80,000 gallons per minute (gpm). The water channels and ponds in the facility contain 13 million gallons of water. If the pump station is operating 5 pumps simultaneously, assuming ideal conditions how long will it take to completely pump the volume of the system through the pump station

Answers

Answer:

t = 32,5 minutes

Step-by-step explanation:

Volume to fill =  13000000 Gal

5 pumps delivering  80000 gal/min

5 * 80000 = 400000 gal/min

If we divide the total volume by the amount of water delivered for the 5 pumps, we get the required time to fill the volume, then

t =  13000000/ 400000

t = 32,5 minutes

kaiden and her brother both open savings accounts. each begin with a balance of zero dollars. for every two dollars that kaiden saves in her account, her brother saves five dollars. if kaiden saves 300 dollars how much did her brother save

Answers

The ratio is 2:5

Kaiden saves=300$Let her brother saves x

ATQ

[tex]\\ \sf\longmapsto \dfrac{300}{x}=\dfrac{2}{5}[/tex]

[tex]\\ \sf\longmapsto 2x=300(5)[/tex]

[tex]\\ \sf\longmapsto 2x=1500[/tex]

[tex]\\ \sf\longmapsto x=\dfrac{1500}{2}[/tex]

[tex]\\ \sf\longmapsto x=750\$[/tex]

Two complementary angles have measures of s and t. if t is less than twice s, which system of linear equations can be used to determine the measure of each angle? Please explain answer. I know s+t=90. But how do you get to t=2s-90

Answers

The required expressions are both equations 1 and 2 as shown:

[tex]s + t = 90 ......... 1[/tex]

[tex]t<2s[/tex] .... 2

Complementary angles are angles that sum up to 90 degrees. For instance, and A and B are complementary if A + B = 90.

According to the question, if two complementary angles have measures of s and t then:

[tex]s + t = 90 ......... 1[/tex]

Twice of 's' is expressed as [tex]2s[/tex]

If t is less than twice s, this can be expressed as [tex]t<2s[/tex] .... 2

The required expressions are both equations 1 and 2 as shown:

[tex]s + t = 90 ......... 1[/tex]

[tex]t<2s[/tex] .... 2

Learn more on word problems leading to simultaneous equations here: https://brainly.com/question/14294864

write two properties of Zero​

Answers

Answer:

the addition properties of zero and multiplication properties of zero

zero is even, not odd not neutral.

zero is neither positive or negative.

At the dog show, there are 4 times as many boxers as spaniels. If there are a total of 30 dogs,how many dogs are spaniels? Plz help me ​

Answers

Answer:

6 spaniels

Step-by-step explanation:

Create 2 equations to represent this, where b is the number of boxers and s is the number of spaniels:

4s = b

s + b = 30

We can plug in 4s as b into the second equation, s + b = 30:

s + b = 30

s + 4s = 30

5s = 30

s = 6

So, there are 6 spaniels.

Solve 2(x - 1) + 3 = x - 3(x + 1) (make sure to type the number only)

Answers

Answer:

x = -1

Step-by-step explanation:

2(x - 1) + 3 = x - 3(x + 1)

Distribute

2x -2+3 = x -3x-3

Combine like terms

2x +1 = -2x-3

Add 2x to each side

2x+1 +2x = -2x-3+2x

4x+1 = -3

Subtract 1 from each side

4x+1-1 = -3-1

4x= -4

Divide by 4

4x/4 = -4/4

x = -1

a cubical water tank is 80 CM broad how many CC of water does it hold​

Answers

Answer:

I think: vol of h20 inside = side³ = (80cm)³ =512000 cm³

Please refer!

Step-by-step explanation:

I'm not sure but hope it helps.

If you invest $ 30 , 700 with an annual interest rate of 8.9 % , compounded daily, how much would you have at the end of 4 years?

Answers

Answer: $43,823.37

Step-by-step explanation:

Formula to calculate the accumulated amount earned on principal (P) at rate of interest (r) compounded daily after t years :

[tex]A=P(1+\dfrac{r}{365})^{365t}[/tex]

As per given , we have

P= $ 30,700

r= 8.9 % = 0.089

t= 4 years

[tex]A=30700(1+\dfrac{0.089}{365})^{365(4)}\\\\=30700(1+0.0002438)^{365(4)}\\\\=30700(1.0002438)^{1460}\\\\=30700(1.42747138525)\\\\=43823.3715272\approx43823.37[/tex]

Hence, the amount at the end of 4 years would be $43,823.37 .

list the domain and range of the relation {(6,8), (7,7), (0,-8), (7,1), (6,8)}

Answers

Answers:

Domain = {0, 6, 7}

Range = {-8, 1, 7, 8}

======================================================

Explanation:

The domain is the set of allowed x inputs. So we simply list the x coordinates. Toss out any duplicates. Sorting the values is optional.

The range is the set of y coordinates. Like with the domain, we'll toss out any duplicates and optionally we can sort the y values.

what is the sum of √-2and√-18

Answers

For this question, we need to simplify some radicals and combine like terms. One thing for sure that should be noticed is the fact that both of these radicals are going to be imaginary, as they both have negatives inside of them.

Let's simplify the radicals:

√-2 =                   ← Note the negative

i√2

√-18 =                  ← Note the negative here as well

i√18 =

i√2·3·3 =

i√2·3² =

3i√2

Now, all we have to do is combine like terms:

i√2 + 3i√2 = 4i√2

Find the value of x to the nearest tenth of a degree.
20.4
21.8
42.9
68.2

Answers

Answer:

Answer is 20.4 .........

The answer is 20.4 because i also got it right

look at the image for the question

Answers

Volume = l × b × h

Length = 7m

Breadth = 7m

Height = 9m

Volume = 7×7×9

441m³

Must click thanks and mark brainliest

for the functions f(x) = 4x^4+4x^3-8x^2-13x-5 and g(x) = x+1, find (f/g)(x) and (f/g)(2)

Answers

Answer:

(f/g)(x) = 4x³ - 8x - 5

(f/g)(2) = 11

Step-by-step explanation:

f(x) = 4x⁴ + 4x³ - 8x² - 13x - 5

g(x) = x + 1

To find (f/g)(2) first find (f/g)(x)

To find (f/g)(x) factorize f(x) first

That's

f(x) = 4x⁴ + 4x³ - 8x² - 13x - 5

f(x) = ( x + 1)( 4x³ - 8x - 5)

So we have

[tex] (f/g)(x) = \frac{( x + 1)( 4x³ - 8x - 5)}{x + 1} [/tex]

Simplify

We have

(f/g)(x) = 4x³ - 8x - 5

To find (f/g)(2) substitute 2 into (f/g)(x)

That's

(f/g)(2) = 4(2)³ - 8(2) - 5

= 4(8) - 16 - 5

= 32 - 16 - 5

= 11

(f/g)(2) = 11

Hope this helps you

x+2y=9 and x=2y+1

helppppppppppppppppp meeeeereeer​

Answers

Answer: y = 2

Step-by-step explanation:

we need to use systems of equations, basically since we know what x equals, we plug that into first equation so:

since x=2y+1,

we can say that:

(2y+1)+2y=9

then we solve this equation,

first we expand the brackets- 2y+1+2y=9,

then we combine like terms- 4y+1=9

then we take 1 away from both sides- 4y=8

then we divided both sides by 4

y=2

Hope this helps xx

Answer:

(5, 2)

Step-by-step explanation:

Given system:

x + 2y = 9x = 2y + 1

Solve for y by substitution:

(2y + 1) + 2y = 94y + 1 = 94y = 8y = 2

Find the value of x:

x = 2*2 + 1x = 5

A rectangular city is 3 miles long and 10 miles wide. What is the distance between opposite corners of the city? The exact distance is ______ miles How far is it to the closest tenth of a mile? Answer: The distance is approximately ______ miles.

Answers

Answer:

The exact distance is [tex]\sqrt{109}[/tex] miles.

The distance is approximately 10.4 miles.

Step-by-step explanation:

It is given that a rectangular city is 3 miles long and 10 miles wide. So,

Length = 3 miles

Width = 10 miles

We need to find the distance between opposite corners of the city. It means, we need to find the length of the diagonal of the rectangle.

Using Pythagoras theorem, the length of diagonal is

[tex]d=\sqrt{l^2+w^2}[/tex]

where, l is length and w is width.

Substitute l=3 and w=10.

[tex]d=\sqrt{(3)^2+(10)^2}[/tex]

[tex]d=\sqrt{9+100}[/tex]

[tex]d=\sqrt{109}[/tex]

The exact distance is [tex]\sqrt{109}[/tex] miles.

Now,

[tex]d=\sqrt{109}[/tex]

[tex]d=10.4403065[/tex]

[tex]d\approx 10.4[/tex]

The distance is approximately 10.4 miles.

Other Questions
Incident action planning and strategic communication planning A. should be contracted to incident action planning specialists during emergency responses. B. are both managed by the planning section chief. C. have no relationship. D. are related because the first step to each is understanding the situation. A divorced woman with 2 young children has a small trust fund that gives her $2,500 a year in income. She collects another $2,500 per year in alimony payments. The woman wishes to make a contribution to an Individual Retirement Account this year. Which statement is TRUE Which OF THESE is a reason why a bird 'sings'?a. To mark out its territory to others of its own speciesb. To frighten other birds that may attack / eat itc. To attract its food like worms and insectsd. To wake up other birds and animalsQuick Can I please have some help with this -> "2.59 describe the composition of the blood: red blood cells, white blood cells, platelets and plasma 2.60 understand the role of plasma in the transport of carbon dioxide, digested food, urea, hormones and heat energy" What was NOT a reason colonists in America staged a revolution against the British government inthe late 1700s?O The British were unfairly taxing the colonists.O The colonists had no representation in the laws that governed them.The British were requiring the colonists to house British troops in their own homes.The colonists felt that the British were over-representing them in parliament. Which of the following Progressive Era legislative achievements was a primary goal of Frances E. Willards activism? A. the Clayton Antitrust Act B. the Eighteenth Amendment C. the Nineteenth Amendment D. the Pure Food and Drug Act Which of the following is not an antioxidant _________1) Sodium benzoate 2) Sulphur dioxide 3) Sulphite salts 4) Citric acid which of the following is a basic function of government? *maintaining a navycreating public parkstaxing the citizensensuring national security what is the product of .4 x .38 i need help please i give 5 stars ;( 0.25 L of aqueous solution contains 0.025g of HCLO4 (strong acid) what will be the Ph of the solution g English is not my native language. How can I test if my language skills are poggers? A cylinder shaped can needs to be constructed to hold 550 cubic centimeters of soup. The material for the sides of the can costs 0.04 cents per square centimeter. The material for the top and bottom of the can need to be thicker, and costs 0.07 cents per square centimeter. Find the dimensions for the can that will minimize production cost. The measure of one of the small angles of a right triangle is 45 less than twice the measure of the other small angle. Find the measure of both angles. A man bought a car for 15000GH cedis . He later sold it at a profit of 25% .What was his selling price. Identify the speaker and context/meaning of the following quote: " I liked old Morrow's mother".Plss no links and this is from Catcher in the Rye chapter 8 A movie theater sells only two types of tickets: adult tickets for $20 each and student tickets for $12 each. One weekend, the theater sold 2,000 tickets for a total of $35,072. How many more adult tickets than student tickets were sold that weekend?A) 768 B) 770 C) 772 D) 774 A car start from rest . If the acceleration of the car is 0.2m/s^2, what will be it's velocity at the end of 3 minutes?And what distance will it cover at that time? Which statement is an example of an opinion?A. The Mexican-American War should be considered one of thegreatest victories of the United States.B. The United States gained large amounts of land because of theMexican-American War.C. The Treaty of Guadalupe Hidalgo, signed in 1848, officially endedthe Mexican-American War.D. The Treaty of Guadalupe Hidalgo established the Rio Grande asthe border between the United States and Mexico.SUBMIT The graph shows the weight of a jar when filled with different numbers of marbles.What does the y-intercept represent?A)The weight of the marbles without the jar.B)The weight of the jar without the marbles.C)The weight of one marble and the jar.D)The unit rate for each marble added.